Maturità 2019: primo problema – secondo punto

Si assuma,d’ora in avanti, di avere a = 1 e b = −1.

  • Studiare le due funzioni così ottenute
  • verificando che il grafico di g ammette un centro di simmetria
  • i grafici di f e g sono tangenti nel punto B(0,−1)
  • determinare inoltre l’area della regione piana S delimitata dai grafici delle funzioni f e g.

Prerequisiti

  • saper fare lo studio di funzione
  • ricordare la condizione per il centro di simmetria di una curva
  • risolvere un sistema uguagliando al parte esponenziale con la parte non esponenziale
  • saper calcolare un integrale definito: area racchiusa tra due curve

Sviluppo

Primo punto

Parto con la f che è una semplice parabola.

Le intersezione con l’asse y ossia ponendo x=0 risulta -1.

L’intersezioni con l’asse x ossia ponendo y=0 e risolvendo l’equazione di secondo grado x^2-x-1=0 sono:

x=\cfrac{1+\sqrt{5}}{2}\simeq 1.6 e

x=\cfrac{1-\sqrt{5}}{2}\simeq -0.6

Con la derivata prima determino il vertice della parabola:

f'(x)=2x-1=0

Il vertice della parabola risulta:

V\left ( \cfrac{1}{2};-\cfrac{5}{4} \right )= V\left ( 0.5; -1.25\right )

è un punto di minimo (basta studiare il segno della derivata prima.

Adesso studio la g.


L’intersezione con l’asse y ossia ponendo x=0 risulta -1.

L’intersezione con l’asse x ossia ponendo y=0 risulta 1.

Studio la derivata prima e di ha dopo alcuni semplici passaggi:

-2x^2+4x-1=0

x_{1}=\cfrac{2+\sqrt{2}}{2}\simeq 1,7

x_{2}=\cfrac{2-\sqrt{2}}{2}\simeq 0,29

Studiando il segno si vede che il primo è il punto di massimo mentre il secondo è il punto di minimo con le seguenti coordinate.

minimo m(0,29;-1.15) e massimo M(1,7;1,16)

La rappresentazione grafica fornisce le seguenti curve:

Secondo punto

Perchè esista un centro di simmetria deve valere la seguente relazione:

\left\{\begin{matrix} x=2a-x'\\  y=2b-y' \end{matrix}\right.

sostituendoli nella g risulta:

2b-y'=(2a-x'-1)e^{2(2a-x')-(2a-x')^2}

e bisogna uguagliarle:

y'=2b-(2a-x'-1)e^{2(2a-x')-(2a-x')^2}=y=(x-1)e^{2x-x^2}

da questa si vede che b=0 e guardando solo i termini che non sono nell’esponente si ha la condizione -2a+1=-1 ossia a=1.

il punto P(1;0) è quello di simmetria. Come si poteva anche vedere dal grafico

Terzo punto

Si deve risolvere il sistema:

\left\{\begin{matrix} y=x^2-x-1\\  y=(x-1)e^{2x-x^2} \end{matrix}\right.

x^2-x-1=(x-1)e^{2x-x^2}

L’esponenziale si deve annullare per cui:

2x-x^2=0 che ha come soluzioni 0 e 2

Anche il polinomi che moltiplica l’esponenziale si deve annullare

x^2-x-1=x-1 che ha come soluzione o e 2.

Quindi si annulla in P(0,-1) e in P'(2;1).

Per verificare la condizione di tangenza si calcola il valore della derivata prima in 0 e deve avere lo stesso valore:

f'(x)=2x-1

f'(0)=-1

g'(x)=(-2x^2+4x-1)e^{2x-x^2}

g'(0)=(-1)e^{0}=-1

ed effettivamente sono tangenti.

Quarto punto

Si deve calcolare l’area della seguente regione di piano:

\int_{0}^{2}(x-1)e^{2x-x^2}-(x^2-x+1)dx

Si deve notare che il primo integrale (x-1) è la derivata a meno di -2 dell’esponente per cui la primitiva risulta:

F(x)=-\cfrac{1}{2}e^{2x-x^2}-\cfrac{x^3}{3}+\cfrac{x^2}{2}+x

F(2)-F(0)=\cfrac{4}{3}.


Pubblicato in Senza categoria | Lascia un commento

Maturità 2019: primo problema – primo punto

Si considerino le seguenti funzioni:

f(x)=ax^2-x+b

g(x)=(ax+b)e^{2x-x^2}

Provare che, comunque siano scelti i valori di a e b in \mathbb{R} con a\neq 0, la funzione g ammette un massimo e un minimo assoluti. Determinare i valori di a e b in corrispondenza dei quali i grafici delle due funzioni f e g si intersecano nel punto A(2,1).

Prerequisiti

  • saper fare la derivata prima
  • segno della derivata prima
  • risolvere un sistema d’equazioni

Sviluppo

Sviluppo la prima parte facendo la derivata prima di g(x).

g'(x)=a\cdot e^{2x-x^2}+(ax+b)e^{2x-x^2}\cdot (2-2x)

annullo la derivata per determinare i potenziali punti di massimo o di minimo

posso eliminare l’esponenziale perchè si annulla solo all’infinito e rimane:

a+(ax+b)\cdot (2-2x)=a+2ax-2ax^{2}+2b-2bx=0

2ax^2-2x(a-b)-a-2b=0

x_{1,2}=\cfrac{(a-b)\pm \sqrt{(a-b)^2+2a^2+4ab}}{2a}=\cfrac{(a-b)\pm \sqrt{2a^2+(a+b)^2}}{2a}

si nota che la quantità sotto la radice è sempre positiva per cui si hanno sempre due valori che potranno essere un massimo ed un minimo.

Per verificare che siano anche massimo o minimo assoluti devo verificare che:

\underset{x\rightarrow \pm\infty }{lim}(ax+b)e^{2x-x^2}=0

\underset{x\rightarrow \pm\infty }{lim}(ax+b)e^{2x-x^2}=\underset{x\rightarrow \pm\infty }{lim}\frac{ax+b}{e^{x^2-2x}}

Applicando De L’Hospital:

\underset{x\rightarrow \pm\infty }{lim}\frac{a}{e^{x^2-2x}(2x-2)}=0

Adesso sviluppo la seconda parte:

Sostituisco le coordinate di A(2,1) in f ed in g e risolvo il relativo sistema:

\left\{\begin{matrix} 1=4a-2+b\\  1=(2a+b)e^{4-4} \end{matrix}\right.

Risolvendola con il metodo che si preferisce si ha:

a=1

b=-1

Pubblicato in Senza categoria | Lascia un commento

Maturità 2019: ottavo quesito

Un protone penetra in una regione di spazio in cui è presente un campo magnetico uniforme di modulo B=1.00mT.

Esso inizia a muoversi descrivendo una traiettoria ad elica cilindrica, con passo costante \Delta x= 38.1 cm, ottenuta dalla composizione di un moto circolare uniforme di raggio r=10.5 cm e di un moto rettilineo uniforme. Determinare il modulo del vettore velocità e l’angolo che esso forma con B.

Prerequisiti

  • conoscenza della forza di Lorentz
  • moto circolare
  • scomposizione dei vettori velocità

Sviluppo

Forza di Lorentz

\overline{\hbox{F}}=q\overline{\hbox{v}}\times \overline{\hbox{B}}

Essendo un moto elicoidale si deve tener presente questo schema:

ossia la formula precedente diventa in modulo

F=qvB\sin \alpha =m\cdot \cfrac{v_{perp}^{2}}{R}

v_{perp}=v\cdot \sin \alpha

per cui quella precedente diventa:

v_{perp}=v\cdot \sin \alpha =\cfrac{qBR}{m}=\cfrac{1,6\cdot 10^{-19}\cdot 1\cdot 10^{-3}\cdot 10,5\cdot 10^{-2}}{1,6\cdot 10^{-27}}=1\cdot 10^{4}\frac{m}{s}

v_{paral}=v\cdot \cos \alpha =\cfrac{\Delta x}{T}

ma il periodo T:

\cfrac{v_{perp}}{R}=\cfrac{2\pi }{T}

T=\cfrac{2\pi R}{v_{perpe}}=65,9\cdot 10^{-4}s

da cui si trova:

v_{paral}=\cfrac{\Delta x}{T}=5,8 \cdot 10^{-3}\frac{m}{s}

v=\sqrt{v_{pepr}^{2}+v_{paral}^{2}}=1,2\cdot 10^{-4}\frac{m}{s}

\tan \alpha =\frac{v_{perp}}{v_{parall}}

\alpha =59.

Pubblicato in Senza categoria | Lascia un commento

Maturità 2019: settimo quesito

In laboratorio si sta osservando il moto di una particella che si nuove nel verso positivo dell’asse x di un sistema di riferimento ad esso solidale.

All’istante iniziale, la particella si trova all’origine e in un intervallo di tempo di 2ns percorre una distanza di 25cm.

Una navicella passa con velocità v=0,80c lungo la direzione x del laboratorio, nel verso positivo, e da essa si osserva il moto della stessa particella.

Determinare le velocità medie della particella nei due sistemi di riferimento.

Quale intervallo di tempo e quale distanza misurerebbe un osservatore posto sulla navicella?

Prerequisiti

  • conoscenza delle trasformazioni di Lorentz
  • addizione delle velocità

Sviluppo

Nel primo sistema di riferimento al velocità media sarà:

v=\cfrac{25\cdot 10^{-2}}{2\cdot 10^{-9}}=1.25\cdot 10^{8}\frac{m}{s}

Nel secondo sistema di riferimento devo applicare la relazione:

v'=\cfrac{v_{p}-v_{n}}{1-\cfrac{v_{n}\cdot v_{p}}{c^{2}}}

v'=\cfrac{1.25\cdot 10^{8}-0.8\cdot 3\cdot 10^{8}}{1-\cfrac{1.25\cdot 10^{8}\cdot 0.8\cdot 3\cdot 10^{8}}{c^{2}}}=-1.73 \cdot 10^{8}\frac{m}{s}

dal sistema della navetta osserverò una contrazione delle lunghezze:

\Delta x'=\cfrac{\Delta x}{\sqrt{1-\cfrac{v^2}{c^2}}}

ossia

\Delta x'=\cfrac{25\cdot 10^{-2}}{\sqrt{1-\cfrac{0.8^{2}\cdot 3\cdot 10^{16}}{c^2}}}=0,1497m

L’intervallo di tempo sarà:

\Delta t'=\cfrac{\Delta x'}{v_{n}-v_{p}^{'}}=\cfrac{0.1497}{0.8\cdot 3\cdot 10^{8}-1.73\cdot 10^{8}}=2.24ns

quindi rispetto al sistema di riferimento della navicella vi è stata una contrazione delle lunghezze ed una dilatazione dei tempi.

Pubblicato in Senza categoria | Lascia un commento

Maturità 2019: quesito sei

Spiegare la relazione esistente tra la variazione del campo che induce la corrente e il verso della corrente indotta. Calcolare la corrente media che passa nella spira durante i seguenti intervalli di tempo:

Una spira di rame, di resistenza R=4,0 m\Omega racchiude un’area di 30cm^{2} ed è immersa in un campo magnetico uniforme, le cui linee di forza sono perpendicolari alla superficie della spira. La component del campo magnetico perpendicolare alla superficie varia nel tempo come indicato in figura.

a: da 0,0 ms a 3,0 ms

b: da 3,0 ms a 5,0 ms

c: da 5,0 ms a 10 ms

Prerequiti

Conoscenza legge di Faraday-Neumann

Sviluppo

f=-\cfrac{d\Phi (B)}{dt}

La corrente

f=Ri

l’unica cosa che varia nel tempo è il campo indizione magnetico secondo il grafico, la superficie rimane costante per cui si avrà:

i=-\cfrac{SB'(t)}{R}

quindi finchè il campo magnetico è negativo si avrà la corrente da un verso appena diventa positivo la corrente avrà verso opposto.

Per calcolare il valore medio della corrente tra i ari intervalli di deve calcolare il valore del campo magnetico in tali intervalli estrapolando i dati.

Intervallo [ms]Campo magnetico B [mT]
0-3B=-\cfrac{2}{90}t^2
3-5B=-\cfrac{2t-8}{10}
5-10B=-\cfrac{-2t+20}{10}

Le funzioni trovate sono: nel primo tratto una parabola con vertice nell’origine e passante per il punto \left ( 3;-\cfrac{2}{10} \right ) negli altri due casi sono delle rette che passano per i punti estratti dal grafico.

Per trovare il valore medio della corrente nei vari intervalli si deve svolgere il seguente integrale:

IntervalloIntegrale
0-3i=-\frac{S}{3R}\int_{0}^{3}B'(t)dt=-\cfrac{S}{3R}\left ( B(3)-B(0) \right )
3-5
i=-\frac{S}{2R}\int_{3}^{5}B'(t)dt=-\cfrac{S}{2R}\left ( B(5)-B(3) \right )
5-10
i=-\frac{S}{5R}\int_{5}^{10}B'(t)dt=-\cfrac{S}{5R}\left ( B(10)-B(5) \right )

Primo intervallo:

i=\cfrac{30\cdot 10^{-4}\cdot 2\cdot 9\cdot 10^{-6}}{3\cdot 4\cdot 10^{-3}\cdot 90}=5\cdot 10^{-8}A

per il secondo ed il terzo è sufficiente sostituire i valori.

Pubblicato in Senza categoria | Lascia un commento

Maturità 2019: quesito cinque

Si lanciano 4 dati con facce numerate da 1 a 6:

  • Qual è la probabilità che la somma dei 4 numeri usciti non superi il 5?
  • Qual è la probabilità che il prodotto dei 4 numeri usciti sia multiplo di 3?
  • Qual è la probabilità che il massimo numero uscito sia 4?

Prerequisito

  • conoscere bene le permutazioni con ripetizione
  • lo schema ad albero per calcolare le probabilità del prodotto e somma di eventi

Sviluppo

Primo punto:

Per avere che la somma non superi il 5 è necessario che si abbia la seguente quaterna:

1 1 1 2

essa può essere vista come una permutazione con ripetizione del gruppo 1 tre volte e quindi le possibili permutazioni sono:

p=\cfrac{4!}{3!}=4

ed anche la quaterna

1 1 1 1

La probabilità che esca 1 è \cfrac{1}{6} come pure che esca il 2 e quindi si deve effettuare il prodotto affinché esca la quaterna 1 1 1 2 ossia

\left ( \cfrac{1}{6} \right )^{4}

siccome può capitare quattro volte, la probabilità cercata sarà:

5\cdot \left ( \cfrac{1}{6} \right )^{4}

Secondo punto:

Il ragionamento è molto simile al precedente

per avere che il prodotto sia un multiplo del 3 si dovranno avere queste quaterne (a fianco inserisco quante possibili permutazioni possono essere inserite e elativa probabilità:

Quaternapermutazioneprobabilità
6 3 3 3 4
4\cdot \left ( \cfrac{1}{6} \right )^{4}
6 6 6 61
\left ( \cfrac{1}{6} \right )^{4}
1 1 1 34
4\cdot \left ( \cfrac{1}{6} \right )^{4}
6 1 1 1 4
4\cdot \left ( \cfrac{1}{6} \right )^{4}
6 6 6 34
4\cdot \left ( \cfrac{1}{6} \right )^{4}
6 6 3 36
6\cdot \left ( \cfrac{1}{6} \right )^{4}

Si sommano adesso tutte le probabilità e si ha:

23\cdot \left ( \cfrac{1}{6} \right )^{4}

terzo punto:

Seguo il ragionamento e lo schema precedente:

quaternapermutazioneprobabilità
4 1 1 14
4\cdot \left ( \cfrac{1}{6} \right )^{4}
4 2 2 244\cdot \left ( \cfrac{1}{6} \right )^{4}
4 3 3 344\cdot \left ( \cfrac{1}{6} \right )^{4}
4 4 4 411\cdot \left ( \cfrac{1}{6} \right )^{4}
4 1 2 266\cdot \left ( \cfrac{1}{6} \right )^{4}
4 1 1 266\cdot \left ( \cfrac{1}{6} \right )^{4}
4 1 3 366\cdot \left ( \cfrac{1}{6} \right )^{4}
4 1 1 366\cdot \left ( \cfrac{1}{6} \right )^{4}
4 2 2 366\cdot \left ( \cfrac{1}{6} \right )^{4}
4 2 3 366\cdot \left ( \cfrac{1}{6} \right )^{4}

Si sommano adesso tutte le probabilità e si ha:

45\cdot \left ( \cfrac{1}{6} \right )^{4}

Pubblicato in Senza categoria | Lascia un commento

Maturità 2019: quarto quesito

Dati i punti A(2,0,-1) e B(-2,2,1), provare che il luogo geometrico dei punti P dello spazio, tali che, \overline{\hbox{PA}}=\sqrt{2}\overline{\hbox{PB}}, è costituito da una superficie sferica S e scrivere la sua equazione cartesiana. Verificare che il punto T(-10,8,7) appartenga ad S e determinare l’equazione del piano tangente in T ad S.

Prerequisiti

  • conoscenza della distanza tra due punti nello spazio
  • definizione di raggio e centro di una sfera
  • conoscenza della retta passante per due punti nello spazio
  • conoscenza della relazione tra vettore direzione della retta e di quello del piano
  • conoscenza condizione di appartenenza di un punto nello spazio.

Sviluppo

Per prima cosa impongo la condizione \overline{\hbox{PA}}=\sqrt{2}\overline{\hbox{PB}}.

\sqrt{(x-2)^2+y^2+(z+1)^2}=\sqrt{2}\cdot \sqrt{(x+2)^2+(y-2)^2+(z-1)^2}

elevando tutto al quadrato e sviluppando i quadrati del binomio si ha:

x^2-x+4+y^2+z^2+2z+1=2x^2+8x+8+2y^2-8y+8+2z^2-4z+2

sommando i monomi simili ed ordinandoli nella forma della sfera si ha:

x^2+y^2+z^2+12x-8y-6z+13=0

per dimostrare che rappresenti una sfera i coefficienti dei termini al quadrato devono essere uguali: ciò è soddisfatto; inoltre il valore del raggio deve dare un valore maggiore di zero. Le coordinate del centro sono:

C(-6,4,3)

e quelle del raggio sono:

r=\sqrt{36+16+9-13}=r=\sqrt{48}

quindi è proprio l’equazione di una sfera:

Per verificare che il punto T(-10,8,7) appartenga alla sfera è sufficiente sostituire le sue coordinate all’equazione della sfera e verificare che si abbia un’identità.

100+64+49-120-64-42+13=0 ed infatti è un’identità:

Adesso determino la retta passante per il centro e per il punto T.

\left\{\begin{matrix} x=(-10+6)t-6\\  y=(8-4)t+4\\  z=(7-3)t+3 \end{matrix}\right.

quindi il vettore direzione ha componenti v(-4,4,4) che sono le stesse componenti del vettore direzione del piano perpendicolare a tale retta.

L’equazione del piano sarà quindi:

-4x+4y+4z-d=0

il piano passa per il punto T, per cui è sufficiente sostituire le sue coordinate per determinare d:

40+32+28+d=0

ed il piano avrà equazione:

-4x+4y+4z-100=0

oppure semplificando:

x-y-z+25=0

Pubblicato in Senza categoria | Lascia un commento

Maturità 2019: terzo quesito

Tra tutti i parallelepipedi rettangoli a base quadrata, con superficie totale di area S, determinare quello per cui la somma delle lunghezze degli spigoli è minima.

Prerequisiti

  • saper sviluppare i problemi di massimo e minimo
  • nozione di base della geometria dei solidi per il calcolo della superficie totale
  • saper calcolare la derivata prima e la relativa disequazione

Sviluppo

Considero x la dimensione della base del mio parallelepipedo, devo esprimere la terza dimensione, che chiamerò a per comodità in funzione di x e dell’altro termine che conosco ossia la superficie totale.

Tutto questo per trovare il min(a+b+c) ma b=c=x per cui risulta:

min(2x+a)

La superfice totale risulta:

S_{t}=2x^{2}+4xa=S

risolvendola tenendo come incognita a:

a=\cfrac{S-2x^2}{4x}

La funzione di cui dovrò calcolare il minimo sarà perciò:

f(x)=2x+\cfrac{S-2x^2}{4x}=\cfrac{6x^2+S}{4x}

facendone la derivata prima si ha:

f'(x)=\cfrac{12x\cdot 4x-4(6x^2+S)}{16x^2}=\cfrac{24x^2-4S}{16x^2}

annullando il numeratore di vede che le soluzioni sono:

x=\pm \sqrt{\cfrac{S}{6}}

ed è positiva per x<-\sqrt{\cfrac{S}{6}} ed x>+\sqrt{\cfrac{S}{6}}

quindi il punto di minimo è x=\sqrt{\cfrac{S}{6}}

da cui si ricavano le tre dimensioni:

b=c=\sqrt{\cfrac{S}{6}}

Pubblicato in Senza categoria | Lascia un commento

Maturità 2019: secondo quesito

E’ assegnata la funzione:

g(x)=\sum_{n=1}^{1010}x^{2n-1}=x+x^3+x^5+\cdot \cdot \cdot+x^{2019}

Provare che esiste un solo x_{0}\in \mathbb{R} tale che g(x_{0})=0. Determinare inoltre:

\underset{x\rightarrow +\infty}{lim}\cfrac{g(x)}{1.1^{x}}.

Prerequisiti

  • definizione di funzione dispari
  • analisi della crescenza e decrescenza di una funzione dalla sua derivata
  • applicazione di De L’Hospital o la conoscenza degli infinitesimi per lo sviluppo del limiti

Sviluppo

La funzione è una funzione dispari ossia:

f(x)=-f(-x)

infatti:

g(-x)=-x-x^3-x^5-\cdot \cdot \cdot -x^{2019}

e

-g(-x)=+x+x^3+x^5+\cdot \cdot \cdot +x^{2019}

quindi è simmetrica rispetto all’origine.

Inoltre facendo la derivata prima di g(x) si ha:

g'(x)=+1+3x^2+5x^4+\cdot \cdot \cdot +2019x^{2018}

che è la somma di soli termini positivi per cui è sempre positiva e quindi la funzione è sempre crescente.

Simmetrica e sempre crescente esisterà solo un punto che la annulla che sarà poi proprio l’origine.

Anche se il grafico non è richiesto il grafico infatti risulta:

Adesso si passa a calcolare il limite.

Siccome si è nella forma infinito su infinito applico De l’Hospital:

\underset{x\rightarrow +\infty}{lim}\cfrac{g(x)}{1.1^{x}}=\underset{x\rightarrow +\infty}{lim}\cfrac{1+3x^2+5x^4+\cdot \cdot \cdot +2019x^{2018}}{1.1^{x}ln(1.1)}.

Iterando De L’Hospital 2018 volte mi troverò un numero diviso 1.1^{x} e quindi il limite assumerà il valore 0.

\underset{x\rightarrow +\infty}{lim}\cfrac{g(x)}{1.1^{x}}=0.

Pubblicato in Senza categoria | Lascia un commento

Maturità 2019: primo quesito

Una data funzione è esprimibile nella forma f(x)=\cfrac{P(x)}{x^2+d} dove d\in \mathbb{R} e p(x) è un polinomio. Il grafico di f interseca l’asse x nei punti di ascisse 0 e \cfrac{12}{5} ed ha come asintoti le rette di equazioni x=3 , x=-3 e y=5. Determinare i punti di massimo e di minimo relativi della funzione f.

Prerequisiti

  • conoscere il significato degli asintoti in relazione alla forma di una funzione
  • conoscere cosa rappresentano i punti che annullano il numeratore
  • conoscere il concetto di derivata per il calcolo dei punti di massimo e di minimo

Sviluppo

Essendoci due asintoti verticali il denominatore si annullerà in 3 e -3.

Si può scrivere nella forma: x^2 -9.

Il numeratore si annulla in 0 e \cfrac{12}{5} per cui può essere scritto come: x\cdot \left ( x-\cfrac{12}{5} \right ); si deve inserire l’ulteriore condizione che è presente un asintoto orizzontale in y=5.

Per avere tale asintoto è sufficiente moltiplicare il numeratore per 5 che risulterà quindi:

5x\cdot \left ( x-\cfrac{12}{5} \right ).

Ricapitolando le affermazioni precedenti la funzione, affinché soddisfi i vincoli dati si può scrivere nella forma:

f(x)=\cfrac{5x^2-12x}{x^2-9}.

Adesso di calcoli la sua derivata prima per la determinazione dei massimi e dei minimi:

f'(x)=\cfrac{(10x-12)(x^2-9)-2x(5x^2-12x)}{\left ( x^2-9 \right )^{2}}=\cfrac{12x^2-90x+108}{\left ( x^2-9 \right )^{2}}

I valori in cui si annulla il numeratore sono 6 e \cfrac{3}{2}.

Studiando il segno della derivata prima si ha che è positiva per x<\cfrac{3}{2} e x>6 per cui il massimo della funzione si ha in x=\cfrac{3}{2} ed il massimo in x=6.

Sostituendo tali valori nella funzione di partenza si avrà il punto di massimo:

M\left ( \cfrac{3}{2};1 \right )

ed il punto di minimo:

m\left ( 6;4 \right )

Il grafico della funzione, anche se non richiesto risulta:

Pubblicato in Senza categoria | Lascia un commento